What is System: Definition and 1000 Discussions

A system is a group of interacting or interrelated elements that act according to a set of rules to form a unified whole. A system, surrounded and influenced by its environment, is described by its boundaries, structure and purpose and expressed in its functioning. Systems are the subjects of study of systems theory.

View More On Wikipedia.org
  1. P

    Generating Function for Lagrangian Invariant System

    Homework Statement Given a system with a Lagrangian ##L(q,\dot{q})## and Hamiltonian ##H=H(q,p)## and that the Lagrangian is invariant under the transformation ##q \rightarrow q+ K(q) ## find the generating function, G. Homework EquationsThe Attempt at a Solution ##\delta q = \{ q,G \} =...
  2. isukatphysics69

    Where Does Thermal Energy Come From in an Inelastic Collision?

    Homework Statement Homework Equations n/a The Attempt at a Solution I am going through the final exam problem sets right now and just looked at the answer key and wondering where in this system there could be thermal energy? while i was working through the problem i assumed no thermal energy...
  3. Y

    Rotational Mast System - Transfer Function, PID Design

    Homework Statement Given the system above, were a motor is rotating a mast that is attached to a second second mast by a spring with a spring constant K and a damping coefficient D. The motor has a current I applied to it with coefficient K Below are the provided values. K_s is the...
  4. M

    How does rotating a coordinate system affect vector direction and components?

    If I move a coordinate system by an angle theta, why does the vector still have the same direction, but the components are different?
  5. R

    System of Two Quantum Particles

    Homework Statement Consider a system of two quantum particles. Each particle has two quantum states, one with zero energy and one with energy ε>0. For each of the three cases, draw a table of the possible microstates α of the system, and find the canonical partition function Z(β). a)The two...
  6. cooldudeachyut

    Friction problem for two block system on an inclined plane

    Homework Statement Two block M1 and M2 rest upon each other on an inclined plane. Coefficient of friction between surfaces are shown. If the angle θ is slowly increased, and M1<M2 then Options : 1- Block A slips first. 2- Block B slips first. 3- Both slip simultaneously. 4- Both remain at...
  7. isukatphysics69

    Ranking the Work on Each System: Which Case Has the Most Positive Work?

    Homework Statement Homework EquationsThe Attempt at a Solution Work = W = ΔKE Case A: ΔKE = WTENSION - WSPRING - WGRAVITY Case B: ΔKE = WGRAVITY - WTENSION Case C: ΔKE - ΔPESPRING = WTENSION - WGRAVITY Case D: ΔKE + ΔPEGRAVITY = -WTENSION Case E: WSYSTEM = 0 Velocity and tension for...
  8. M

    A Fredholm scattering theory to quantum dissipative system

    Dear Community, I am trying to figure out what is happening in this article (https://journals.aps.org/prb/abstract/10.1103/PhysRevB.29.130) when they are calculating the Fredholm Determinant (Section IV). The basic idea is that you want to solve $$ k = |\frac{det(1+h_0)}{det'(1+h_0+v)}| $$...
  9. Michael Sullivan

    ABCD matrix for an optical system of two thick lenses

    Homework Statement I have an object at distance x1 from the first thick lens(convex) then air at distance x2 to the next thick lens(concave) then air of distance x3 to a mirror. I need to build an ABCD matrix representing this. Homework Equations thick lens equation: [ A B ] = [ 1-d/R1...
  10. Andy Bloch

    I Momentum w/o Net Velocity? Exploring the Lorentz Factor

    (I hope this post doesn't cross the border into the forbidden realm of quackery and speculation.) I have what seems like a simple question about Special Relativity but I haven’t seen it discussed anywhere, nor has anyone I've asked. Does the nonlinearity of the Lorentz factor provide a way...
  11. Danny Boy

    A Information of system vs system, apparatus and environment

    Suppose we have a quantum system ##Q## with an initial state ##\rho^{(Q)}##. The measurement process will involve two additional quantum systems: an apparatus system ##A## and an environment system ##E##. We suppose that the system ##Q## is initially prepared in the state ##\rho_{k}^{(Q)}## with...
  12. T

    Normal modes of a system of springs

    I'm looking at what should be just a simple spring system where four identical springs are holding up a square, load-bearing pallet plate in a warehouse. Now, someone says the equation of motion for the vertical normal mode of vibration is simply d2z/dt2 = -4(k/m)z. Right away however, I see no...
  13. M

    A Explaining the Geocentric Celestial Reference System

    Hi there guys, I was wondering does anyone have a layman's explanation of the GCRS as defined in the title. I am confused as to whether this is an inertial or non inertial system. In text modern reference books such as this (chapter 10, section 10.3.2) they define rotating/non rotating...
  14. R

    Calculating pressure within a system at a specific location?

    Homework Statement Solution: Homework Equations dp/dz = -ρh Pressure varies linearly with the depth. The Attempt at a Solution Firstly, I have calculated the pressure at point B. PB = PA - (0.3m)*(1000kg/m^3)*(9.81m/s^2) PB = 88057 Pa Calculating the pressure at point C. However, my...
  15. CDL

    Composite Spin 1/2 System Probability Question

    Homework Statement Two spin ##\frac 12## particles form a composite system. Spin A is in the eigenstate ##s_z = + \frac 12## and Spin B is in the eigenstate ##s_x = + \frac 12## What is the probability that a measurement of the total spin will give the value ##0##? Homework Equations I know...
  16. R

    MHB Explaining Number Systems to Students: Ideas & Solutions

    I need to explain to a student the meaning of number systems. I try to explain him that it is like a family of numbers that have some proprity. If one have an idea to expalin him the term, it will be helpful. I remind note about the term: A number system can be complex system number, rational...
  17. BiGyElLoWhAt

    Other Research Proposal For Senior thesis - GR Solar System Model

    Hi All. On Tuesday I have to present my research proposal for my undergrad research. I intend to model our solar system using general relativity and compare and contrast my results with observations. Attached is my paper that I had to submit to my adviser. He suggested that I talk about some...
  18. P

    Moment of inertia of a system of masses

    Homework Statement The moment of inertia about an axis through the center of mass of a system consisting of two masses 3.0 and 5.0 kg connected by a rod of negligible mass 0.8m long is Homework Equations [/B] I = I1 + I2 The Attempt at a Solution I added all the moments of inertia of the...
  19. M

    Free body diagram of a preloaded system with a gasket

    Hello,Ive got a question and wondering if there is someone that could help me? I would really appreciate it! I am studying for an exam that I am going to have in a couple of weeks and it has been going great besides that I have been stuck on a question about tension in a preloaded system with a...
  20. B

    Linear Algebra System of Equations/Rates Application Help

    Homework Statement Suppose that we have a system consisting of two interconnected tanks, each containing a brine solution. Tank A contains x(t) pounds of salt in 200 gallons of brine, and tank B contains y(t) pounds of salt in 300 gallons of brine. The mixture in each tank is kept uniform by...
  21. L

    How to calculate the pressure on a piston in a damper system?

    Hello all, I'm trying to calculate the pressures exerted back on a piston by a working hydraulic fluid in a damper system. I've attached a drawing of the system and the equations I'm using. Essentially the piston motion is restricted by flow through an orifice. I have data for the flow rates...
  22. Maria

    Simple closed force system -- Mechanics

    Homework Statement There is a system of 4 points located along a circle of radius R. Points are connected by undeformable ropes (pink on the picture). There is a force applied to each point. Scheme: https://we.tl/MsCEViCQdB I need to find resulting force in the system. The Attempt at a...
  23. Toreno

    Calculating orbits of planets in Solar System

    Hi, I am about to write a simple application, simulating in 3D current location of the planets in Solar system. Honestly, I do not know even how to start. Can you please guide me where could I find some good information on planet orbits, current locations and how to make all of the calculations...
  24. Theia

    MHB Looking for a serie solution for a nonlinear ODE system

    Hi! \begin{cases} \dot{q} = a \left( 1 - q^2 \right) \\ \dot{a} = - \alpha - a^2 q\end{cases} \qquad \alpha \in (0, 1 ) I've looked into this ODE system about 7 months now, but I've not got anything promising how to write down the solution. I'm mostly interested in q-serie. (To those of you...
  25. adamaero

    System turns on alarm once temp. dips below 10.5°C

    Homework Statement Homework Equations (Above given. I think it's a hint.) The Attempt at a Solution How is delta T = 0.25?? Where is the 10kΩ coming from? We were given a solution for this homework, since it's not collected. It just seems out of order.
  26. incredibe1999

    Maple ODE System Solution Maple: Plotting Earth's Orbit with Sun at Origin

    Hey there! I was trying to plot a ODE solution, but am not getting what I should be. It is actually plotting the orbit of Earth with Sun at the origin. My equations The (-0.00011847) is GM. The Initial Conditions: The plot I get: Should not I be getting a elliptic/circular plot as the...
  27. thegreengineer

    Control of an inverted pendulum (or any other dynamic system in general), how to implement it?

    Good afternoon people. I wanted to implement a physical system using control just to put into practice what I have learned in school. To be more specific: I would like to implement an inverted pendulum like the one on the photo In this system a control input (a force u) drives the cart (mass...
  28. D

    Change in the Kinetic Energy of a System

    The question is to find the change in kinetic energy of a system. the system in use is a pully system with a wooden block on a flat surface attached to a 100g mass. I have calculated force of friction between the block and desk, and using a ticker timer, I have individual speeds at every 0.1 of...
  29. Antarres

    Null curve coordinate system

    So, I've been studying some tensor calculus for general theory of relativity, and I was reading d'Inverno's book, so out of all exercises in this area(which I all solved), this 6.30. exercise is causing quite some problems, so far. Moreover, I couldn't find anything relevant on the internet that...
  30. Physics345

    Increasing Yield of a System with Le Chatelier's Principle

    Homework Statement I am given 2NH3(g) + 2.5O2(g) ⇌ 2NO(g) + 3H2O(g)+heat. By using Le Chatelier's principle, describe and explain four ways in which an industrial chemist could increase the yield of nitrous oxide. Homework EquationsThe Attempt at a Solution For 2NH3(g) + 2.5O2(g) ⇌ 2NO(g)...
  31. _maxim_

    Making a system with mobile electrical contacts

    Hi all, want to realize a system for a PCB which holds a solenoid coil moving in a vertical axis long about 600 mm. The coil must get electric contact only in some predefined positions so that, for a given quote, the coil connects to a different LC circuit for having different resonant frequency...
  32. W

    Rotating mirror system for shifting a beam?

    (sorry if posting in the wrong place) I am looking for some mirror setup where rotation one or more elements in the mirror setup will allow to shift the beam along its axis. Two mirrors in a galvanometer scanner type setup work well for controlling the angle of the beam. Rotating risley...
  33. J

    I Causal Fermion System and revival of Dirac Sea

    If the Higgs Field could exist with constant 246GeV across all of space. How come the Dirac Sea couldn't exist? If the Universe can easily accommodate Higgs Field.. why not Dirac Sea for all particles. Also how does the Dirac Sea of bosons work? Like W+, W-? Any idea? I was asking about the...
  34. T

    MHB Homogeneous, underdetermined equation system

    Hi! Just started with linear algebra Could someone help me with this problem? 2x_1 + x_2 - x_3 + 3x_4 - 3x_5 = 0\\ 3x_1 + 2x_2 + x_3 + 2x_4 + 2x_5 = 0\\ -4x_1 + 3x_2 + 2x_3 + x_4 - 4x_5 = 0 (Sorry, I don't know how to do these big brackets for equation systems in Latex.) So it's a...
  35. R

    Finding a maximum force in a beam type system before failure

    Homework Statement See attached photo Homework Equations Stress = F/A Area = pi d^3 / 4 Sum F h = 0, Sum F v = 0, Sum M = 0 The Attempt at a Solution Hey everyone, after reading the question a number of times and not knowing for certain where to start, I figured because the stress and area...
  36. Jayalk97

    Linearizing a system of equations?

    Homework Statement So these are the equations of motion for a quad-copter. I am supposed to create a MATLAB model for the z-axis. In order to do this I have to linearize the equations around these points, and arrange them in state space representation. Homework Equations As above The...
  37. A

    Binary Stellar System: 15M⊙ & 10M⊙ Revisited

    Homework Statement A binary stellar system is made of one star with ##M_1=15{M}_\odot## and a second star with ##M_2=10{M}_\odot## revolving around circular orbits at a relative distance of ##d=0.001pc##. At some point ##M_1## explodes in a supernovae leaving a neutron star of mass...
  38. H

    A Stability for a system of nonlinear ODEs

    Hi, I am looking at the following system of ODEs: \begin{eqnarray*} \dot{\omega}_{3}+\alpha\omega_{3} & = & \frac{\beta_{1}+\beta_{3}}{\rho_{0}}J_{3} \\ \dot{J_{3}}+2(\alpha_{2}-\alpha_{1})\beta_{2} & = & 0 \\ \dot{\beta}_{1}+\omega_{3}\beta_{2} & = & 0 \\...
  39. Noora Alameri

    I Two dimenstional Heisenberg Hamiltonian for spin 1/2 system

    Hey everybody, I am trying to expand a system of seven qubits from one dimensional Hamiltonian to the two dimensional representation. I have the one dimensional representation and I don't know what to add to transform it from 1D to 2D representation. I would really appreciate your help and...
  40. N

    System of Differential Equations, Phase Plane

    Homework Statement I am working through problem #1, a-c. Homework Equations The main equations are dx/dt=Ax, (A-rI)v=0, and det(A-rI)=0. The Attempt at a Solution [/B] Here is my attempt. I am fairly confident in my answer to A. I'm less sure on my answer to B, however it is the same as...
  41. L

    A Can disjoint states be relevant for the same quantum system?

    In the algebraic approach, a quantum system has associated to it one ##\ast##-algebra ##\mathscr{A}## generated by its observables and a state is a positive and normalized linear functional ##\omega : \mathscr{A}\to \mathbb{C}##. Given the state ##\omega## we can consider the GNS construction...
  42. A

    I Cavitation in a closed loop pumping system

    Dears Kindly, I want to hear from you regarding this topic. The system is closed cooling system. I believe it is difficult to attribute the sound you hear from centrifugal pump to be a cavitation especially if the discharge pressure is normal. The last time we had a problem with this system...
  43. Blueberrymuffim

    The tension in the cord in this system

    Homework Statement A force F is applied horizontally to a block A of mass mA = 0.600 kg so that it slides on the floor with acceleration g/3. Block B of mass mB = 0.200 kg rests on top of A, but is kept in place by a cord which is tied to the wall at 45.0°. The coefficient of kinetic friction...
  44. A

    Binary system of stars (##\alpha## - centauri)

    Homework Statement ##\alpha##-Centuary is in a binary visual system with another star. Their separation, from their CM, is 8.0'' and 9.7''. The distance from the Earth is 1.31pc. Their revolution period around the CM is 80.1 years. I have to find masses and luminosities for each star. Homework...
  45. S

    Period of a mass spring system with 2 spring of same K(vert)

    A mass attached to a spring is oscillating in Simple Harmonic Motion. If an other spring of same sprinc constant is attached parrallel to the other spring, what is the period of this new system (as a function of the initial period). Here's what I did and have no idea if this is right: For the...
  46. R

    Creating system of equations from word problem optimization

    I have this word problem, and was wondering how I would go about creating a system of equations. Here is the question: Problem: You are a small forest landowner, and decide you want to sustainably harvest some of timber on your property. There are costs related to the infrastructure needed to...
  47. M

    How Does Total Energy Relate to Potential Energy in a Closed System?

    Homework Statement Why is it that when the total energy of a system less than the max potential energy, it must equal to some potential energy in a potential energy diagram. Homework EquationsThe Attempt at a Solution Total energy intersects multiple points in U(x) diagram. However, since...
  48. dRic2

    I How small does a system have to be for QM to be necessary?

    If we are talking bout a single atom QM is obviously necessary, but when we are talking about a bunch of molecules (like gases/fluids) statistical mechanics works fine too. I remember it has something to do with ##\hbar## but I don't remember much and I can not quantify it. Thanks
  49. PlanetGazer8350

    Calculate iron container acceleration in railgun system on the Moon

    Having a cylindrical iron container with wall thickness of 20 cm, and a total weight of 500 tons when filled with its cargo, how would you be able to calculate its required voltage and current input in a vertical railgun system (relative to the Moon's surface), with an exit acceleration of...
  50. F

    Pumping Water -- Please check my pump system design work

    Hey Folks! Just need a quick sanity check ... Putting in a new 4in well and submersible pump ... trying to wrap my brain around the "physics" of it ... Here is what I ***Think I Know*** : 1) The Energy required to move any given volume of water to ground level can be calculated using M*G*H ...
Back
Top